AAMC CBT3 and 3R OFFICIAL Q&A

This forum made possible through the generous support of SDN members, donors, and sponsors. Thank you.

Vihsadas

No summer
Moderator Emeritus
Lifetime Donor
10+ Year Member
5+ Year Member
15+ Year Member
Joined
Oct 17, 2007
Messages
5,474
Reaction score
56
This is the official Q&A thread for AAMC CBT3 and 3R.

Please post ONLY questions pertaining to AAMC CBT3 and 3R.
Out of respect for people who may not have completed the other exams, do not post questions or material from any other AAMC exam.

Please see this thread for the rules of order before you post.

Good luck on your MCAT!

Members don't see this ad.
 
Can someone help with #144?

I really wanted to pick B, but it says the spindle microtubules are "not assembling" which I took to be "not there!" .... so how would mitosis happen, let alone two sets of chromosomes in daughter cells? >.<
 
Can someone help with #144?

I really wanted to pick B, but it says the spindle microtubules are "not assembling" which I took to be "not there!" .... so how would mitosis happen, let alone two sets of chromosomes in daughter cells? >.<

The drug preventing the microtubule spindle formation does not mean mitosis will not happen. It just means it won't happen completely, i.e. the chromosomes will still line up at the mid-plate and you will have the cell arrest in between prophase-metaphase since the chromosomes cannot be pulled apart. Thus, the daughter cell will have more than 1 set of chromosomes since they were not pulled apart by the microtubules.
 
The drug preventing the microtubule spindle formation does not mean mitosis will not happen. It just means it won't happen completely, i.e. the chromosomes will still line up at the mid-plate and you will have the cell arrest in between prophase-metaphase since the chromosomes cannot be pulled apart. Thus, the daughter cell will have more than 1 set of chromosomes since they were not pulled apart by the microtubules.
But won't things freeze up there and thus no anaphase / cytokinesis ?
 
Members don't see this ad :)
But won't things freeze up there and thus no anaphase / cytokinesis ?

Well, I believe telophase happens via contractile elements which is microfilaments/myosin. Regardless of the division or not, if you have a cell that arrests in prometaphase due to the inability of the spindle complex to form and separate the chromosomes you will end up with a cell that has, "more than two sets of chromosomes".
 
Verbal #73
"According to the passage, magnetic fields are primarily byproducts of ____"

Answer: electric currents

But a line from the passage is "dynamo theory, which holds that the magnetic field results from fluid motions in the outer core"

How do you know to choose "electric currents" instead of "fluid motion"?
 
Verbal #79
"According to the passage, a good program would be described as "

Answer: involves viewers just enough to keep them in their chairs [lol I would have never chosen this one]
My Answer: features passionate and articulate characters.

In the last sentence of the passage, the author criticizes TV for lacking passion. So shouldn't the author want passion?

I also thought the author wanted articulate characters (the author dislikes how TV uses inarticulate characters).
 
can someone explain how we are supposed to decide between aromaticity and sterics as the answer to the question? both decrease nucleophilicity...


Screen_Shot_2014_08_19_at_5_24_16_PM.png
 
can someone explain how we are supposed to decide between aromaticity and sterics as the answer to the question? both decrease nucleophilicity...


Screen_Shot_2014_08_19_at_5_24_16_PM.png


I got this question right. TBR states how a primary amine is such a good nucleophile that it immediately reacts to become secondary then tertiary, at which point it is too sterically hindered to react. I just applied that fact to the question.
 
My turn.

I really don't understand this answer. TBR explains how the most insoluble ions are precipitated out first in an experiment, yet this answer choice is telling me the opposite. I mean I COULD'VE used solubility rules to figure this one out quickly, (and get the right answer), but I was trying to be as close to the passage as possible. What gives? What did you guys do to solve this problem?


EDIT: Answer on first page. Sorry.
 
Last edited:
Yeah, I originally got that one wrong, too. So, here is my best shot at explaining it.

In short, in the first step Pb(SO4) (Compound A) precipitated after Na2SO4 was added to aqueous Pb(NO3)2. This tells us Pb(SO4) is less soluble then Pb(NO3)2.

The second step involved adding KI and agitating the solution until some Pb(SO4) (Compound A) was dissolved. The result was that PbI precipitated, so PbI2 is less soluble than Pb(SO4).

In the third step PbI2 was then mixed with Na2CO3 and all of the PbI2 was converted to PbCO3, which precipitated out of solution. So, This tells us that PbCO3 is less soluble than PbI2.

Given that, if we now know the following in terms of solubility:

PbCO3 < PbI2 < Pb(SO4) < Pb(NO3)2

So, on to the answers:

(a) SO4 then I --- No, from our trend above you can see that PbI is less soluble than Pb(SO4), so I would precipitate as PbI2 BEFORE SO4 would precipitated as PbSO4
(b) CO3 then I -- Correct order
(c) SO4 then CO3 -- No, see explanation to (a)
(d) I then CO3 -- No, see explanation to (a)
@Romz

I got this question wrong too. Since you seem to understand it now, do you mind explaining how the stuff in the procedure that precipitates out later is considered LESS soluble? The post to which you were referring (see above) doesn't really go into details about that.

It just doesn't seem intuitive to me. If something is really that insoluble, it should precipitate first.
 
BS #44

2 SO2 + O2 ---> 2SO3

I think this reaction is exothermic (since entropy is negative, deltaH must be negative).

So when the question asks for how to increase the yield of SO3, shouldn't reducing the temperature cause a shift to the RIGHT (making more SO3)?

AAMC says reducing the temperature would not work to increase yield of SO3.
 
@Romz

I got this question wrong too. Since you seem to understand it now, do you mind explaining how the stuff in the procedure that precipitates out later is considered LESS soluble? The post to which you were referring (see above) doesn't really go into details about that.

It just doesn't seem intuitive to me. If something is really that insoluble, it should precipitate first.

Not a problem! Took me long to understand it as well.
If you've got your TBR next to you, flip to and look at Pages 195-197 of the first book of General Chemistry.

The principle they're applying here is that seen in Ion exchange chromatography and its actually a very subtle nuance.

WHY I GOT THIS QUESTION WRONG:

I thought we had a solution of our anions and we were adding lead (Pb 2+) and watching for which one would precipitate out first. THIS IS WRONG. Re-read the experiment again.

WHY THIS QUESTION IS RIGHT:

Upon re-reading the experiment again, we'll notice that they were sequentially reacting the products with a new set of reactants to see which one would form a precipitate. For example, in reaction #1-->#2, they DO NOT completely filter away Reaction Compound A. Instead, they react it with KI to form a new precipitate PbI. That is to say:

PbSO4 (Compound A) + KI <----> PbI2 + Whatever. Looking at this reaction, we see that between PbSO4 and PbI2, PbI2 is less soluble than PbSO4, hence it forms a precipitate. This chain of reactions continues downstream until the experiment ends. (PbI2 more soluble than PbCO3). Hence CO3 is the least soluble. To refer back to my earlier suggestion, this is the theory behind ion exchange chromatography, in which we elute the more soluble compound initially attached in the matrix through a metathesis reaction to form a more insolube compound not containing our desired anion/cation.

I hope this makes it more clear. IMO this is a GREAT MCAT passage testing our ability to really understand what's going on rather than hastily applying theory. They fooled me too - hopefully it doesn't happen again :p.
 
  • Like
Reactions: 1 user
what is the definition of analog? The osteoporosis passage mentions it a lot but I don't quite understand what it means.
 
Members don't see this ad :)
what is the definition of analog? The osteoporosis passage mentions it a lot but I don't quite understand what it means.
From the passage I kind of seemed to get the vibe that it meant "substitutes" but I wouldn't worry too much cause none of the questions in that passage ask about it. I feel like they put it there just to throw us off.
 
what is the definition of analog? The osteoporosis passage mentions it a lot but I don't quite understand what it means.

From the passage I kind of seemed to get the vibe that it meant "substitutes" but I wouldn't worry too much cause none of the questions in that passage ask about it. I feel like they put it there just to throw us off.

Analogs are things that behave similarly because of similar chemical structure. There are many analogs of the most common nucleotide bases. Some of them are simply halogenated.
 
So I completely botched PS passage IV about harmonics. I could not figure out which of the lines corresponded to first, second, third? I was looking for a legend but there wasn't any. It doesn't clearly say but the number of periods in the wave correspond to the harmonic?
 
Last edited:
I am confused about ps section #23. I put "in phase". The correct answer makes sense too but why can't it be A?
 
I am confused about ps section #23. I put "in phase". The correct answer makes sense too but why can't it be A?
I feel like you are trying to over think it. The question refers to figure1a, while perhaps in phase could be a possibility, figure1a explicitly shows the relationship of displacement and time. Since they all intersect at 0, then they all have the same zero displacement.
 
  • Like
Reactions: 1 users
So I completely botched PS passage IV about harmonics. I could not figure out which of the lines corresponded to first, second, third? I was looking for a legend but there wasn't any. It doesn't clearly say but the number of periods in the wave correspond to the harmonic?

Hey I don't know if you wrapped this up from your other post but I saw this and thought I would reply - only FL I have taken so far!

It says the figure has the first three harmonics - the lowest frequency will be the first harmonic; this is the solid line - we can see it has the longest period.

The wave with the peak right behind the n=1 wave is the n=2, and the highest frequency wave is n=3.

You can tell just by seeing which wave has a peak first - and determine that is the highest harmonic.
 
Hey I don't know if you wrapped this up from your other post but I saw this and thought I would reply - only FL I have taken so far!

It says the figure has the first three harmonics - the lowest frequency will be the first harmonic; this is the solid line - we can see it has the longest period.

The wave with the peak right behind the n=1 wave is the n=2, and the highest frequency wave is n=3.

You can tell just by seeing which wave has a peak first - and determine that is the highest harmonic.
Yeah I posted here first but then I sort of understood it so I wanted to make sure I was not wrong lol


So you have only taken aamc3?
 
Haha. I actually really enjoyed that passage. It was so much better than trying to answer questions about Li.
 
Haha. I actually really enjoyed that passage. It was so much better than trying to answer questions about Li.
Haha yeah it wasn't so bad, missed one. The Li passage though :confused: I got 3 wrong. Also I was running out of time on the last 3 passages. I am so bad at timing. How did you do on the VR ?
 
For verbal #66 how can you conclude that the artists wanted financial support from the ruling class?
 
I missed that one also.

Looking back at my test I see I was able to cross out choices A and C leaving it between financial support and "only types of experiences worth recording".

In retrospect I think that financial support was the best answer because they wouldn't have tried to paint other classes if the choice I made was right.

I don't think there was a direct answer from passage other than ruling out other choices and inferring the answer from the fact that the artists "served" the wealthy.
 
For verbal confucius passage, can someone explain #84?

Also for #85, I understand that A is the better answer, but for C, why is people responding to force or strategies irrelevant? The question mentions force or strategies so at first I thought it was.
 
Question about VR #56:

I choose C: Most Americans do not take the legal status of a substance into account when deciding whether or not to ingest it.

The credit response was B: Most Americans are unlikely to engage in an obviously dangerous activity.

While I narrowed it down to those two choices, why would B be a better choice than C? I was thinking if...

[Well as I am typing this I am starting to see why I was wrong...] The question stem talks about a dramatic increase in drug abuse. So if it is really dangerous people will not start to begin with (no dramatic increase.) While choice C is stating that they are currently using; not about future users. If I am missing something, please enlightenment me. Thanks.
 
Question about VR #56:

I choose C: Most Americans do not take the legal status of a substance into account when deciding whether or not to ingest it.

The credit response was B: Most Americans are unlikely to engage in an obviously dangerous activity.

While I narrowed it down to those two choices, why would B be a better choice than C? I was thinking if...

[Well as I am typing this I am starting to see why I was wrong...] The question stem talks about a dramatic increase in drug abuse. So if it is really dangerous people will not start to begin with (no dramatic increase.) While choice C is stating that they are currently using; not about future users. If I am missing something, please enlightenment me. Thanks.
C is a pretty strong statement to make (that people don't care about breaking the law) that doesn't have any support in the passage, whereas the passage (as I recall – too lazy to pull up the test) does explicitly talk about the idea that most people wouldn't do drugs no matter what since they're dangerous.
 
C is a pretty strong statement to make (that people don't care about breaking the law) that doesn't have any support in the passage, whereas the passage (as I recall – too lazy to pull up the test) does explicitly talk about the idea that most people wouldn't do drugs no matter what since they're dangerous.

Thanks, I appreciate that.
 
Could someone please help me with #40. It reads:
What is the change in enthalpy for the reaction below at 25 oC

2HCl(g)----->H2(g) +Cl2(g)
Hf at 25oC:HCl(g)=-92.5 kJ/mol

I understand how you multiply by 2 to get an enthalpy of 185.0 kJ; however, the answer description says how the question asks for the enthalpy change of the reverse reaction? How do I know that the reaction is reversed, because the answer is +185.0kJ

Thank you
 
Last edited:
@Dominique8604

You are given the enthalpy of formation for HCl and it should be prior knowledge that the enthalpy of formation for elements in their standard states is 0.

The formula is dHf = dHf (Products) - dHf (Reactants) = dHf (H2) + dHf (Cl2) - 2[dHf (HCl)] = 0 + 0 - 2(-92.5) = +185

I also see their explanation talks about reversing the reaction, I find that reasoning slightly more confusing if someone is not familiar with thermodynamic cycles.
 
  • Like
Reactions: 1 users
Could someone please help me with the following question. Its #50. It reads:

In the molecule shown below, which bond is the longest. I really have no clue how to decipher this one.
 
@Dominique8604

There are two ways to tackle that problem.

The short way is to see that bonds C and D are equivalent and both are answer choices, so they can be ruled out. Then H has such a small Bohr radius that it's bond length is shorter than an O-N bond.

The proper way is to draw the structure out. If you do this you will see that the structure is a Zwitterion with a double bond between N and one of the two right-most Oxygens. This has a resonance form that gives each of the right N-O bonds partial double bond character and double bonds are shorter than single bonds.
 
  • Like
Reactions: 1 user
Can someone explain Question 81 (AAMC 3R)? It is a VR/CARS strengthen Q.
I am confused why C is wrong.

Thanks
 
Top